Quantcast
Channel: Integral $\int_0^1\frac{x^9\left(x^4+x^2-x-1-5\ln x\right)}{\left(x^{10}-1\right)\ln x}\mathrm dx$ - Mathematics Stack Exchange
Viewing all articles
Browse latest Browse all 2

Integral $\int_0^1\frac{x^9\left(x^4+x^2-x-1-5\ln x\right)}{\left(x^{10}-1\right)\ln x}\mathrm dx$

$
0
0

A friend of mine sent me an integral that she had not been able to crack, and me neither. It comes with a result, but without a proof (I suppose it originated in some math contest). Could you please suggest an approach to prove the result?

$$\int_0^1\frac{x^9\left(x^4+x^2-x-1-5\ln x\right)}{\left(x^{10}-1\right)\ln x}\mathrm dx=\frac12\gamma+\frac{11}5\ln2-\frac54\ln5+\frac12\ln\pi-\frac12\ln\phi,$$where $\gamma$ is the Euler–Mascheroni constant, and $\phi$ is the golden ratio.


Viewing all articles
Browse latest Browse all 2

Latest Images

Trending Articles





Latest Images